Double sommation

Bonjour ou bonsoir (voir l'heure)
Calculer la somme : $$

A_n= \sum_{i=1}^n \sum_{j=1}^n \max(i,j).

$$ Merci d'avance.

Réponses

  • Bonjour,

    Un enfant de cinq ans sait le calculer. Quel âge as-tu ? Que trouves-tu ?
  • :-D . On appelle cela obstacle épistémologique.

    Je sais faire quelques séries, mais je ne sais pas faire celle-ci.
  • $$A_n= \frac{n^3.(n+1)}{2}.

    $$ Est-ce que c'est la réponse ?
  • C'est faux
    Le 😄 Farceur


  • $@gebrane$

    Oui je sais , que c'est faux , car elle utilise la somme $\sum_1^n K^2$ , mais je ne sais pas ou ?
  • Puisque l'enfant ( de 5ans) de notre cher YVesM sait le faire, prend ton courage pour le faire
    Le 😄 Farceur


  • Pour $(i,j)$ entre $1$ et $n$, il est clair que $\max(i,j)$ est entre $1$ et $n$.
    Pour $k$ entre $1$ et $n$, combien de couples $(i,j)$ satisfont à $\max(i,j)=k$ ?
    • parmi les couples tels que $i\le j$, pour lesquels $\max(i,j)=j=k$, il y en a $k$ (correspondant aux $i$ de $1$ à $k$) ;
    • parmi les couples tels que $i>j$, pour lesquels $\max(i,j)=i=k$, il y en a $k-1$ (correspondant aux $j$ de $1$ à $k-1$) ;
    en tout, cela fait donc $2k-1$ couples pour chaque $k$. D'où la valeur de ta somme : \[S_n=\sum_{k=1}^n(2k-1)k=2\sum_{k=1}^nk^2-\sum_{k=1}^nk=\frac{n(n+1)(2n+1)}3-\frac{n(n+1)}{2}=\frac{n(n+1)(4n-1)}{6}.\]
  • Quand je fais un calcul compliqué, comme celui de MathCoss, à la fin, j'essaie de vérifier. Ca prend quelques secondes, et ça permet parfois de détecter des erreurs.
    Si n=0, on devrait trouver 0... Oui :)
    Si n=1, on devrait trouver 1... Oui :)
    Si n=2, on devrait trouver 7... Oui :)
    A priori, MathCoss ne s'est pas trompé.
    Tu me dis, j'oublie. Tu m'enseignes, je me souviens. Tu m'impliques, j'apprends. Benjamin Franklin
  • Vu que c'est un polynôme de degré trois, il aurait vraiment valu la peine de faire un test de plus : quatre valeurs pour un tel polynôme, c'est presque la garantie de la justesse ! Tiens, je le fais moi-même. In:
    sage: def M(n):
    ....:     return [max(i+1,j+1) for i in range(n) for j in range(n)]
    ....: 
    sage: [(add(M(k)), k*(k+1)*(4*k-1)/6) for k in range(1,5)]
    [(1, 1), (7, 7), (22, 22), (50, 50)]
    
  • Bonsoir,
    est-ce que ça ne serait pas une pyramide de cubes ?
    Ça aide peut-être de visualiser ceci comme une pile de carrés de tailles décroissantes à laquelle on a ôté une tranche verticale de côté ?

    edit on enlève une tranche verticale parce-que sinon on compte une diagonale 1 fois de trop, pour chaque carré.
  • Une de plus intéressante et troublante
    $$ \sum_{i=1}^n \sum_{j=1}^n \max(i-1,j+1).$$
    Le 😄 Farceur


  • Un calcul mécanique des premiers termes et une enquête auprès de l'OEIS suggère que le résultat est $\dfrac{2}{3} \, n^{3} + \dfrac{1}{2} \, n^{2} + \dfrac{11}{6} \, n - 1$. En quoi est-ce « intéressant et troublant » ?
  • Bonjour Cher Math Coss,


    Le mot intéressant est relatif, je suis d'accord
    Troublant car wolphi suggère deux cas https://www.wolframalpha.com/input/?i=\sum_{i=1}^n++\sum_{j=1}^n+\max(i-1,j+1).
    Mais en reprenant mon courage, les vérifications pour n=1 ou 2 ou 3 montre que ta formule reste valable ( qui est la deuxième de wolphi)
    Le 😄 Farceur


  • $@Math \; Coss$

    Grand merci cher $Math \; Coss$
  • D'accord, je vois ce que tu veux dire. En effet, c'est curieusement formulé. Cela illustre que si on fixe seulement trois valeurs $(y_1,y_2,y_3)$, on peut trouver plusieurs polynômes $p$ de degré $3$ tels que $p(i)=y_i$ pour $1\le i\le 3$.
  • Math Coss, quand même quelle curieuse comédie vous jouez, vous savez bien que qu'un polynôme de degré 3 sera déterminé par 4 valeurs distinctes (la conditions est suffisante ... (...)) ici la vraie question serait de savoir qu'est est le degré du polynôme générique qui donne la solution au problème de gebrane
    3 ou 4 messages au dessus

    car le "presque" d'un de vos message est si ironique. (et nous semblons savoir que la réponse est 3, pour 4 indéterminées)
  • Bah, pas si ironique. Je voulais juste signaler que la vérification proposée par lourrran aurait pu devenir une démonstration en ajoutant un calcul du même genre et un argument pour le polynôme de degré trois. Quant à la réponse à gebrane, elle marquait un vrai étonnement face à la complication inutile de la réponse de Wolfram alpha.
  • alors je pose la question: en conjecturant qu' une une d-sommation (soit la représentation de d signes $\Sigma$) est est un polynôme de degré $d+1$ ou $2^d$, le polynôme est de degré $d+1$ ou $2^d$
    je sens bien que la réponse sera la première option.
  • La question est un peu vague. Ce qui est vrai et bien connu, c'est qu'il existe pour tout $p$ un polynôme de degré $p+1$ appelé polynôme de Bernoulli (unique) tel que \[\sum_{k=0}^nk^p=\frac{B_{p+1}(n)-B_{p+1}(0)}{p+1}.\]Les premières valeurs de ces sommes sont classiques :
    • $\displaystyle\sum_{k=0}^nk^0=n+1$ ;
    • $\displaystyle\sum_{k=0}^nk^1=\frac{n(n+1)}{2}$ ;
    • $\displaystyle\sum_{k=0}^nk^2=\frac{n(n+1)(2n+1)}{6}$ ;
    • $\displaystyle\sum_{k=0}^nk^3=\frac{n^2(n+1)^2}{4}$.
  • Les sommes polynomiales sont données par un polynôme de degré supérieur de 1, pour les min et les max, comment déterminer le degré ?
    C'est là que je pose la question et je reconnais que j'ai été vague.
  • A priori, rien d'évident dans le fait qu'une somme où interviennent des min et des max soit polynomiale. Dans ce cas très précis, on peut par exemple arguer comme ici sans pousser le calcul jusqu'au bout.

    Avec ce type d'arguments, on peut montrer que si $u_{n+1}-u_n$ est un polynôme de degré $d$ en $n$, alors $u_n$ est un polynôme de degré $d+1$. Cela peut te donner une façon d'attaquer la somme de gebrane.
  • pour la somme de gebrane j'aime bien la méthode géométrique de découpage de domaine en zones, avec des cubres des tours, des étages, des carrés, et tout ce la échelonnés bref je sais que c'est une analogie poussée à l'excès et comme d'habitude et sans calcul (mon gros défaut en math avec la paresse qui le concurrence bien)

    mais je sais aussi que d'utiliser le degré de nilpotence de l'opérateur sur l'espace des suites dont l'expression est polynomiale suivant: $u= (u_n)_{n \in \mathbf{N}} \mapsto v=(v_n)_{n \in \mathbf{N}} \; s.a \; v_n=u_{n+1}-u_{n} \; n>0$
    est la bonne façon de faire les choses.
  • @callipiger
    On est intéressé par ta méthode géométrique. Des détails ?
    Le 😄 Farceur


  • bon je ferai le truc géométrique demain car je ne l'ai pas encore formalisé assez. et surtout que je viens de trouver une autre idée plus simple à formaliser:
    pour les max on peut s'en sortir en se ramenant a des cas connus avec la formule $max(i,j)=\frac {i+j+|i-j|}{2}$
    je regarde la matrice $(i+j)_{\leq 1 i,j \leq n}$ et la matrice $(|i-j|)_{\leq 1 i,j \leq n}$
    qui sont respectivement des matrices constante pas diagonale et constante par "antidiagonale"

    en gros je fait la moyenne d'une tour dont le coins NW vaut 2 , le coin SE vaut $2N$ et les coins SW et NE valent $N+1$
    et d'une autre tour dont le toit est en creux: la diagonale est nulle et les sous diagonales croissent et décroissent jusqu'aux points SW et NE ou la hauteur est du toit est $N-1$

    pour la suite on suppose que N est pair pour simplifier
    (sinon le point central est compté plusieurs fois(4), et les lignes passant par le centre de l'immeuble (NS) et (EW) 2 fois ... "c'est comme un diagrammes de Venn (non sphérique i.e pas de recoupement E-W ou N-S, sauf si les 4 sont là simultanément, ou un tableau de Mondrian) mais avec 4 carrés , donc 9 zones")

    dans le cas pair il y a 4 zones, définissant 4 sous immeubles dont les toits sont des plans.
    et dans les cas où le toit est un plan la somme est le fruit d'une transformation affine

    finalement en écrivant mon idée différente j'ai fini par écrire la méthode géométrique dans le cas du problème de départ
  • Je vous révèle ma méthode pour calculer la double somme initiale $\sum_{i=1}^n \sum_{j=1}^n \max(i,j)$
    C'est une méthode matricielle. Soit $A$ la matrice définie par $a_{i,j}=\max(i,j)$, la matrice s’écrit $A=\begin{pmatrix}
    1 & 2 & 3 & \dots & n\\
    2 & 2 & 3 & \dots & n\\
    3 & 3 & 3 & \dots & n\\
    \vdots & \vdots & \vdots & \ddots & \vdots\\
    n& n & n & \dots & n
    \end{pmatrix}$
    La somme cherchée est un jeu d'enfant de 5 ans.
    Le 😄 Farceur


  • $@ Tout \; le \; monde $

    :-S Les deux chiffres que j'ai compris , s'affiche $virus \; detected$
  • @ gebrane sur cette méthode matricielle je vois une pyramide
    je pense que l'on fait la même chose mais visualisé/exprimé différemment.
  • \begin{align*}
    \sum_{i=1}^{n} \sum_{j=1}^{n}\max(i,j)&= \sum_{i=1}^{n}(\sum_{j=1}^{i}(\max(i,j)+ \sum_{j=i+1}^{n}\max(i,j))\\
    &= \sum_{i=1}^{n}(i^{2}+\frac{(n-i)(n+i+1)}{2} )\\
    &=\sum_{i=1}^{n}i^{2}+ \sum_{i=1}^{n}\frac{(n-i)(n+i+1)}{2} \\
    &=\frac{n(n+1)(4n-1)}{6}
    \end{align*}
  • Merci Keynes
    Le 😄 Farceur


  • Bonsoir
    en partant de la matrice de gebrane, on reconnaît immédiatement que c'est un cube de côté $n$ auquel on a ôté une pyramide de hauteur $n-1$
    donc partant de là vérifions par le calcul si je retombe sur mes pattes.
    \begin{align*}
    n^3-\sum_{k=0}^{n-1}k^2&=n^3-\frac{n(n-1)(2(n-1)+1)}{ 6}=n\big(n^2-\frac{(n-1)(2n-1)}{6}\big) \\
    &=n\big(\frac{6n^2-2n^2+3n-1}{6}\big)=n\frac{(n+1)(4n-1)}{6}.

    \end{align*} Je retombe sur les résultats annoncés à plusieurs reprises, je suis content de moi donc X:-(
    Bon je repars sur l'autre sujet.
  • callipiger écrivait : je suis content de moi

    Tu n'as pas honte, c'est ma matrice, je connais mes droits
    je t'interdis de l'utiliser sans mon consentement (:D
    Le 😄 Farceur


  • gebrane si tu le permets... m'autoriserais-tu à voir la matrice qui correspond à ton problème alternatif ?
  • Je te la donne ; elle est à toi mais il faut savoir l’écrire
    Le 😄 Farceur


  • $@Keynes$

    Belle preuve $Keynes$
  • J'ai calculé le $\min (i;j)$ $$


    \sum_{i=1}^n \sum_{j=1}^n \min(i,j)=\frac{n(n+1)(2n+1)}{6}.

    $$ Vrai ou faux ?
    Remarque :
    \begin{align*}
    \sum_{i=1}^n \sum_{j=1}^n \min(i,j)&=\frac{n(n+1)(2n+1)}{6}= \sum_{i=1}^n k^2 \\

    \sum_{i=1}^n \sum_{j=1}^n ij &= \frac{n^2(n+1)^2}{4}= \sum_{i=1}^n k^3

    \end{align*} Est-ce que c'est normal ça ?
  • On peut voir que pour tout $n\geq 0$, $A_n = n^3 - \sum_{k=1}^{n-1} k^2$ sur un dessin (dessiner le graphe de la fonction $i,j\mapsto \max(i,j)$ et regarder ce qui manque pour former un cube d'arête $n$), autrement dit $A_n=n^3-\frac{(n-1)n(2n-1)}{6}$
    Une fonction est un ensemble $f$ de couples tel que pour tous $x,y,z$, si $(x,y)\in f$ et $(x,z)\in f$ alors $y = z$.
Connectez-vous ou Inscrivez-vous pour répondre.